u and v are position vectors with terminal points at (-8, 5) and (-3, -12), respectively. Find the terminal point of u + v

(-11, -7)
(-11, 7)
(-5, -7)
(5, -17)

Answers

Answer 1

Answer:

(-11,-7)

Step-by-step explanation:

 U           V

(-8,5) + (-3,-12)

(-8 - 3) , (5 - 12)

-11 , -7

(-11,7)

Answer 2

Answer:

(-11,-7)

Step-by-step explanation:

u =  (-8, 5)

v =  (-3, -12)

u+v = ( -8+-3, 5+-12)

      = (-11,-7)


Related Questions

find ''m'' and ''c'' of 3-2x+y=0 by comparing with y=mx+c

Answers

Answer:

m=-2 c=3

Step-by-step explanation:

y=mx+c

rearrange

-mx-c+y=0

original

y=2x-3

Glass A measures 84 mm in diameter and 175 mm tall. Glass B measures 96 mm in diameter and 125 mm tall. Which glass holds more liquid? How much more?

Answers

Answer:

Glass A

Step-by-step explanation:

Volume = п r ² h

п = 3.14  aprox.

r = radius = diameter/2

h = tall

glass A

volume = 3.14 * (84/2)² * 175

volume = 3.14 * 42² * 175

volume = 3.14 * 1764 * 175

volume = 969318mm³

glass B

volume = 3.14 * (96/2)² * 125

volume = 3.14* 48² * 125

volume = 3.14 * 2304 * 125

volume = 904320mm³

Answer:

969318 > 904320

then:

Glass A holds more liquid

the product of -9/17 and its reciprocal​

Answers

the product of -9/17 and it's reciprocal will be 1

32
Two forces one is 10N and other is 6N act on a body The directions are unknown the resultant force on the body is
a. between 4 and 16N
b. more than 6N I
c. more than 1ON
d between 6 and 16N

Answers

If Forces are acting on opposite direction

[tex]\\ \rm\hookrightarrow F_{net}=F_2-F_1[/tex]

[tex]\\ \rm\hookrightarrow F_{net}=10-6[/tex]

[tex]\\ \rm\hookrightarrow F_{net}=4N[/tex]

If both acting on same direction

[tex]\\ \rm\hookrightarrow F_{net}=F_1+F_2[/tex]

[tex]\\ \rm\hookrightarrow F_{net}=10+6[/tex]

[tex]\\ \rm\hookrightarrow F_{net}=16N[/tex]

Hence

[tex]\boxed{\sf 4N\leqslant F_{net}\leqslant 16N}[/tex]

Please help me I will give brainlist

Answers

Answer:

please send the whole question.

actually the value of f(x) is missing

Rachelle buys a drink. She spends the same amount of money on her drink as Chuck spent on his candy. Rachelle now has only 1/3 of the same amount of money that she had before she bought the drink. How much money did Rachelle have before she bought the drink?

Answers

Answer:

$1.80

Step-by-step explanation:

2/3x = 1.2

x = 1.2 / (2/3)

x = 1.2 * (3/2)

x = 1.80

what do you think would happen to a person if mortor neuron from the grey matter of spinal cord is removed?​

Answers

Step-by-step explanation:

Introduction

Until World War II, a serious spinal cord injury (SCI) usually meant certain death. Anyone who survived such injury relied on a wheelchair for mobility in a world with few accommodations and faced an ongoing struggle to survive secondary complications such as breathing problems, blood clots, kidney failure, and pressure sores. By the middle of the twentieth century, new antibiotics and novel approaches to preventing and treating bed sores and urinary tract infections revolutionized care after spinal cord injury. This greatly expanded life expectancy and required new strategies to maintain the health of people living with chronic paralysis. New standards of care for treating spinal cord injuries were established: reposition the spine, fix the bones in place to prevent further damage, and rehabilitate disabilities with exercise.

Today, improved emergency care for people with spinal cord injuries, antibiotics to treat infections, and aggressive rehabilitation can minimize damage to the nervous system and restore function to varying degrees. Advances in research are giving doctors and people living with SCI hope that spinal cord injuries will eventually be repairable. With new surgical techniques and developments in spinal nerve regeneration, cell replacement, neuroprotection, and neurorehabilitation, the future for spinal cord injury survivors looks brighter than ever.

Consider the equation: x 2 − 6 = 2 − 18 x x 2 −6=2−18xx, squared, minus, 6, equals, 2, minus, 18, x 1) Rewrite the equation by completing the square. Your equation should look like ( x + c ) 2 = d (x+c) 2 =dleft parenthesis, x, plus, c, right parenthesis, squared, equals, d or ( x − c ) 2 = d (x−c) 2 =dleft parenthesis, x, minus, c, right parenthesis, squared, equals, d. 2) What are the solutions to the equation? Choose 1 answer: Choose 1 answer: (Choice A) A x = 9 ± 89 x=9±89x, equals, 9, plus minus, 89 (Choice B) B x = − 9 ± 89 x=−9±89x, equals, minus, 9, plus minus, 89 (Choice C) C x = 9 ± 89 x=9± 89 ​ x, equals, 9, plus minus, square root of, 89, end square root (Choice D) D x = − 9 ± 89 x=−9± 89 ​ x, equals, minus, 9, plus minus, square root of, 89, end square root

Answers

Answer:

1. (x+9)^2 = 89

2. (Choice D) D x = − 9 ± 89 x=−9± 89 ​ x, equals, minus, 9, plus minus, square root of, 89, end square root

Step-by-step explanation:

x^2 - 6 = 2 - 18x

1) rewrite the equation by completing the square

x^2 - 6 = 2 - 18x

x^2 + 18x = 2+6

x^2 + 18x = 8

Find the half of the coefficient of x and square it

18x

Half=9

Square half=(9)^2

=81

Add 81 to both sides

x^2 + 18x = 8

x^2 + 18x + 81 = 8 + 81

x^2 + 18x + 81 = 89

(x+9)^2 = 89

Check:

(x+9)(x+9)=89

x^2 + 9x + 9x + 81=89

x^2 + 18x +81 =89

2) (x+9)^2 = 89

√(x+9)^2 = √89

x+9=√89

x=√89 - 9

It can be rewritten as

x= -9 ± √89

(Choice D) D x = − 9 ± 89 x=−9± 89 ​ x, equals, minus, 9, plus minus, square root of, 89, end square root

Make the biggest possible number using the digits below only once 0 , 3 , 4

Answers

Answer:

12,157,665,459,057,000,000

Step-by-step explanation:

It seems like it would have to involve exponents.  3 to the 40th power would be 12,157,665,459,057,000,000.  

The digits of a 2 digit number differ by 3. Is the digits are interchanged, and the resulting number is added to the original number, we get 143. What can be the number?

Answers

Answer:

58

Step-by-step explanation:

Hello, let's note the two digits a and b. the first number 'ab' can be written as 10a +b. For instance if this is 24 it can be written 20 + 4.

If the digits are interchanged the number become 'ba' so 10b + a

We can say that 10a + b + 10b + a = 143

11(a+b)=143

We divide by 13 both sides and we take

a+b = 143/11 = 13

and we know that the digits differ by 3 so b = a + 3

then a + b = a + 3 + a = 2a + 3 = 13

so 2a = 10 and then a = 5

Finally, b = 5+3=8 so the number is 58.

And we can verify that 58 + 85 = 143.

Thanks

Answer:

Let the unit digit be x and tens digit be x + 3

Therefore, the original number = 10(x + 3) + x

On interchanging, the number formed = 10x + x + 3

❍ According to Question now,

➥ 10(x + 3) + x + 10x + x + 3 = 143

➥ 10x + 30 + 12x + 3 = 143

➥ 22x + 33 = 143

➥ 22x = 143 - 33

➥ 22x = 110

➥ x = 110/22

x = 5

__________________...

Therefore,

The unit digit number = x = 5

The tens digit number = x + 3 = 5 + 3 = 8

__________________...

The original number = 10(x + 3) + x

The original number = 10(5 + 3) + 5

The original number = 50 + 30 + 5

The original number = 85

Hence,the original number is 85.

7x-y= 22
4x + 2y = 10
can someone help plz?

Answers

Answer:

x = 3

y = -1

Step-by-step explanation:

Based on the fact that our have two variables and two equations, I'm going to assume you are trying to find the values of x and y.

For these two equations, let's first find the value of x using the equation elimination method.

We're going to eliminate the y variable by multiplying the first equation by 2 and then adding that new equation to the second equation.

7x - y = 22 ==> 14x - 2y = 44

4x + 2y + (14x - 2y) = 10 + (44)

18x + 0y = 54

x = 3

Now we plug the value of x back into one of the equations to find the value of y.

7x - y = 22

7(3) - y = 22

-y = 22 - 21

y = -1

So our values are as follows:

x = 3

y = -1

Cheers.

At Geri’s school, 2 out of every 9 eighth-grade girls tried out for the volleyball team. If there are 216 eighth-grade girls at the school, how many tried out for the team?

Answers

Answer:

48 girls tried out for the team.

Step-by-step explanation:

The phrase "2 out of every 9" can be written as the fraction 2/9. To find our answer, all we have to do is 2/9 * 216 = 48 girls.

Sonia works at a bakery. The function f(x) represents the amount of money Sonia earns per loaf, where x is the number of loaves she makes. The function g(x) represents the number of bread loaves Sonia bakes per hour, where x is the number of hours she works. Show all work to find f(g(x)), and explain what f(g(x)) represents.

f(x)=9x^2+1
g(x)=square root 2x^3

Answers

Answer:

18x^3+1

Step-by-step explanation:

since g(x)=√2x^3 and f(x)=9x^2+1 then

f(g(x))= 9(g(x))^2 +1 = 9(√2x^3)^2 +1 = 18x^3+1

this represents the amount of money Sonia earns baking loaves in x hours

The equation of f(g(x)) is [tex]f(g(x)) = 9(\sqrt{2x^3})^2 + 1[/tex], and it represents the amount of money Sonia makes when she bakes for x hours

The functions are given as:

[tex]f(x) = 9x^2 + 1[/tex]

[tex]g(x) = \sqrt{2x^3}[/tex]

Recall that:

[tex]f(x) = 9x^2 + 1[/tex]

Substitute g(x) for x

[tex]f(g(x)) = 9(g(x))^2 + 1[/tex]

Substitute [tex]g(x) = \sqrt{2x^3}[/tex]

[tex]f(g(x)) = 9(\sqrt{2x^3})^2 + 1[/tex]

Hence, the equation of f(g(x)) is [tex]f(g(x)) = 9(\sqrt{2x^3})^2 + 1[/tex]

Read more about composite functions at:

https://brainly.com/question/10687170

What is the solution to the system of linear equations? Write you answer as a coordinate. 2x − y = 4 5x + 2y = 10 Show All Work !!

Answers

Answer:

(2, 0)

Step-by-step explanation:

2x - y = 4

5x + 2y = 10

Solve by elimination by multiplying the top equation by 2, so the y values will cancel out (-2y + 2y = 0)

4x - 2y = 8

5x + 2y = 10

Add them together and solve for x:

9x = 18

x = 2

Plug in 2 as x in one of the equations to find y:

2x - y = 4

2(2) - y = 4

4 - y = 4

-y = 0

y = 0

The solution is (2, 0)

Help? Hello Help me please

Answers

Answer:

  $562.75

Step-by-step explanation:

The future value formula is good for this.

  FV = P(1 +r/n)^(nt)

where principal P is invested at annual rate r compounded n times per year for t years.

Using your numbers, we find the account value to be ...

  FV = $500(1 +.06/2)^(2·2) = $500·1.03^4 ≈ $562.75

There will be $562.75 in the account after 2 years.

A, B are the points (-4, -3) and (2, 5). If the interval AB is now divided into 4
equal parts, find the coordinates of these three points.

Answers

Answer:

D(-2,5;1); C(-1,1); E(0.5,3).

Step-by-step explanation:

Imagine that there is a line AB, it is divided into 4 equal parts. If there are 4 parts, There are 4+1=5 points. That's obvious that if parts are equal , the third point is on the middle of the line AB, suppose it is the point C

xc= (xa+xb)/2

xc=( -4+2)/2= -1

yc= (-3+5)/2=1

(-1,1)

There are to unknown points D (between A and C), and E (between C and B). D is on the middle of AC .

Xd= (-4-1)/2=-2.5

yd= (-3+1)/2=-1

(-2,5;1)

E is on the middle of CB

xe= (-1+2)/2= 0.5

ye=(1+5)/2=3

(0.5, 3)

plz help, explain me the method
Topic: Simultaneous equations
Class 9 ICSE​

Answers

Answer:

heyy matee...

going according to given parameters

let 2 numbers be x and y

so atq =>

3x + y = 142 => y = 142 - 3x

and 4x - y = 148

4x - ( 142 - 3x ) = 138

4x - 142 + 3x = 138

7x = 280

x = 40

now y = 142 - 3(40) = 142 - 120 = 22

so x = 40 and y = 22

Answer:

Hope so, it helped you.

Step-by-step explanation:

For solving the equation i elimanated y by adding both the equation.

Solve for x: two thirds plus one third times x equals two times x
NEED ANSWER ASAP DUE INN THREE HOURS

Answers

Hey there! I'm happy to help!

Let's write this equation out.

2/3+1/3x=2x

We want to get all of the variables on the left side and all the numbers on the right side so we can see what number x is equal to. First, Let's subtract 2/x from both sides.

2/3+1/3x-2x=0

We subtract 2/3 from both sides.

1/3x-2x=-2/3

We combine our x values.

-5/3x=-2/3

We divide both sides by -5/3.

x=-5

Have a wonderful day and keep on learning! :D

On the coordinate plane below, Point P is located at (2,-3), and point Q is located at (-4,4). Find the distance between points P and Q Round your answer to the nearest whole number.

Answers

Answer:

9

Step-by-step explanation:

When given two points (x₁, y₁) and (x₂, y₂) on a plane, the Formula for the distance between the points is calculated as:

D= √(x₂ - x₁)² +(y₂ - y₁)²

In the question, we are given Point P is located at (2,-3), and point Q is located at (-4,4).

P(2, -3) , Q(-4, 4)

D = √(-4 - 2)² +(4 -(-3))²

D = √(-6)² + (7)²

D =√36 + 49

D = √85

D = 9.2195444573

Approximately to the nearest whole number, the distance between points P and Q is 9

Please answer it now

Answers

Answer:

x = 19

Step-by-step explanation:

The angles of a triangle add to 180

U and S are equal since they are the base angles  and UT and ST are equal

S + T + U = 180

x + 26 + 90 + x+26 = 180

Combine like terms

2x +142 = 180

Subtract 142 from each side

2x = 38

Divide each side by 2

2x/2 = 38/2

x = 19

Answer:

19

Step-by-step explanation:

x+26+x+26+90 = 180

2x + 52 = 90

2x = 38

x = 19

On Tuesday, Dec. 3, I began drinking a glass of cola every day except Saturday and Sunday. I drank my 22nd glass of cold on A) Dec. 24 B) Dec. 25 C) Dec. 31 D) Jan. 1

Answers

Answer:

The correct option is;

D) Jan. 1

Step-by-step explanation:

The given information are;

The date at which drinking a glass of cola a day of cola began = Dec 3

The days in which to drink cols = Every day of the week except Saturday and Sunday

The number of glasses of drinking cola = 22

In the fires week, number of days in which to drink cola = Tuesday, Wednesday, Thursday, and Friday which is 4 days

On the week commencing Dec 9, 5 glasses drank

On the week commencing Dec 16, 5 glasses drank

On the week commencing Dec 23, 5 glasses drank

On the week commencing Dec 30, 3 glasses drank

Therefore on the week commencing Dec 30, cola was drank on the 30th, 31st and the 22nd glass was drank on Jan. 1

The correct option is Jan. 1.

The number that is 75% of one less than a number n.

Answers

Answer:

The number is n-0.75

Step-by-step explanation:

Here in this question, we are interested in giving a number which is 75% of 1 less than a number n.

The first thing we do here is to calculate the value of 75% of 1.

That would be;

75/100 * 1 = 0.75

So this value less than n will be;

n- 0.75

Question 9(Multiple Choice Worth 4 points)
(02.05 LC)
Which of the following statements best describes the effect of replacing the
graph of y = f(x) with the graph of y = f(x - 9)?
The graph of y = f(x) will shift up 9 units.
The graph of y = f(x) will shift down 9 units.
The graph of y = f(x) will shift left 9 units.
The graph of y = f(x) will shift right 9 units.
una

Answers

Answer:

The graph will shift 9 units to the right

Sorry about my hand writing

Answer: It moves 9 units to the right

Step-by-step explanation:

It moves 9 units to the right is because if it's inside parentheses it moves right/left instead of up/down. I also talked to my Algebra Teacher about it and got it right in the final test.

PLS HELP ME ON THIS QUESTION I WILL MARK YOU AS BRAINLIEST IF YOU KNOW THE ANSWER!!
Find the median of the data set that consists of 10, 10, 8, 4, 5, 7, 7, 4, 3, 10.
A. 10
B. 7.5
C. 7
D. 6.8

Answers

Answer:

7

Step-by-step explanation:

the median is the middle value of the list. first off, write the list of numbers in ascending order (small to big)

3, 4, 4, 5, 7, 7, 8, 10, 10, 10

since there are 10 values, there are two middle numbers which are 7 and 7. since they are both 7, the median is 7

Please help me understand this number sequence

Answers

Answer:

Step-by-step explanation:

A=a(r)^t

a=1

time=2.5 hours=25/10 ×60=150 minutes

10t=150

t=150/10=15

[tex]A=1(2)^{15}=32,768[/tex]

Help, I need this answered soon

Answers

Answer:

9.

Step-by-step explanation:

45 / [8(5 - 4) - 3]

= 45 / (8 * 1 - 3)

= 45 / (8 - 3)

= 45 / 5

= 9.

Hope this helps!

9

Work:
PEMDAS
45/8 (5 - 4) - 3
45/8 (1) - 3
45/8 - 3
45/5
9

Brad invests $3700 in an account paying 3% compounded monthly. How much is in the account after 8 months?

Answers

Answer:

Amount after 8 month (A) = $3775 (Approx)

Step-by-step explanation:

Given:

Amount invested (P) = $3,700

Rate of interest (r) = 3% = 0.03 / 12 = 0.0025 monthly

Number of month (n) = 8 month

Find:

Amount after 8 month (A)

Computation:

[tex]A=P(1+r)^n\\\\ A=3700(1+0.0025)^8\\\\A=3700(1.02017588)\\\\ A = 3774.650676[/tex]

Amount after 8 month (A) = $3775 (Approx)

Giving brainliest!!!! Plzz put the correct answers.
2^(10)= 2x...x2 how many times
15^(57)= 15x...x15 how many times
(-4)x...x(-4) 7 times =
(1.5)x...x(1.5) 12 times =
If you give me the answer after like an hour i willl report you!!

Answers

Answer:

See below

Step-by-step explanation:

aⁿ = a×a×a×....×a  (power n of the number a  =  number a multiplied by itself n times)

2^(10)= 2x...x2 how many times  =  10 times 2

15^(57)= 15x...x15 how many times  = 57 times 15

(-4)x...x(-4) 7 times =  (-4)^(7)

(1.5)x...x(1.5) 12 times = (1.5)^(12)

A highway measuring 90 feet x 7 feet requires 1/2 a fluid ounce of cleaning situation per square root how much cleaning situation is required to clean the hallway

Answers

Answer:

1260 ounce of the fluid.

Step-by-step explanation:

Dimension of hallway = 90 feet × 7 feet

Area of the hallway = 90 × 7

                                  = 630 square feet

Given that 1/2 (0.5) of a fluid ounce is required per square foot, the amount of cleaning situation to clean the hallway can be determined as;

                 =  [tex]\frac{area of hallway}{cleaning situation per foot}[/tex]

                 = [tex]\frac{630}{0.5}[/tex]

                 = 1260 ounce

The amount of cleaning situation required to clean the hallway is 1260 ounce of the fluid.

The sum of a rational number and an irrational number is irrational.

Always true,sometimes true, never true

Answers

Answer:   Always true

=========================================================

Explanation:

We can prove this by contradiction.

Let's say

A = some rational numberB = some irrational numberC = some other rational number

and

A+B = C

We'll show that a contradiction happens based on this.

If A is rational, then A = p/q where p,q are two integers. The q cannot be zero.

If C is rational, then C = r/s for some other integers. We can't have s be zero.

Note the following

A+B = C

B = C - A

B = r/s - p/q

B = qr/qs - ps/qs

B = (qr - ps)/qs

B = (some integer)/(some other integer)

This shows B is rational. But this is where the contradiction happens: We stated earlier that B was irrational. A number cannot be both rational and irrational at the same time. The very definition "irrational" literally means "not rational".

In short, I've shown that if A+B = C such that A,C are rational, then B must be rational as well.

The template is

rational + rational = rational

Therefore, we've shown that if A is rational and B is irrational, then C cannot possibly be rational. C is irrational.

Another template is

rational + irrational = irrational

Other Questions
2sin^2(2x) + 1 = 3sin(2x) Solve for x with exact answers. The domain is 0 x The pH scale is used to tell if a substanceis an acid or base. Substances with a pHof 7 are neutral. An acid is anythingbelow 7 and a base is anything above 7.Bleach has a pH of about 12. What type ofsubstance is bleach?A. baseB. acidC. neutral Do you think that companies operating around the Globe can really do without job description? Why or Why Not? You can use examples to support your arguments what is the definition of patoralism Two sides of a triangle are 24 inches in length, what is the length of the third side heeeellllp..................... find the measure of c Mr. Blake's biology class is divided into three secrions. The same test is given to each section. The table above shows both the lowest score and the range of scores on this rest for each section What is the overall range of all scores in all three sections? The hourly wage increase each employee receives each year depends on their number of years of service. Every three years of service means an increase of $0.50 per hour. So, employees that have been with the company for less than three years can expect to receive an increase of $0.50 per hour. Employees that have been with the company for at least three years, but less than six years can expect an increase of $1.00. Employees that have been with the company for at six years, but less than nine years, receive an increase of $1.50 per hour. And, employees of at least nine years, but less than twelve years receive an increase of $2.00. Write a function to represent this scenario.Which graph represents this wage increase for x < 12? Who is responsible for responding to workflow(s) for equipment dispatch requests through the business workplace require An approving authority must approve The table below shows the results from a study that compared speed (in miles per hour) and average fuel economy (in miler per gallon) for cars. Find a quadratic model for the data.0.008y=13.472x 2 +0.746x0.008y=25.836x+0.049y=25.836x+0.049y=.008x2+0.746x+13.472y=.008x 2 +0.746x+13.472y=0.049x+25.836y=0.049x+25.836 Select the correct answer. What is the mood of this excerpt from "The Open Boat" by Stephen Crane? The January water was icy, and he reflected immediately that it was colder than he had expected to find it off the coast of Florida. This appeared to his dazed mind as a fact important enough to be noted at the time. The coldness of the water was sad; it was tragic. This fact was somehow so mixed and confused with his opinion of his own situation that it seemed almost a proper reason for tears. The water was cold. A. overwhelmed B. optimistic C. condescending D. grateful E. expectant Bella Pool Company sells prefabricated pools that cost $80,000 to customers for $144,000. The sales price includes an installation fee, which is valued at $20,000. The fair value of the pool is $128,000. The installation is considered a separate performance obligation and is expected to take 3 months to complete. The transaction price allocated to the pool and the installation is Explain why each is WHS task is important and provide examples of procedures that staff could follow to ensure safety.1.infection control2.housekeeping3.signage 5 examples of run ons Assume all variables represent non negative numbers . Help please!! Assist Pleaseshow work What must writers do when using language taken directly from an outside source? Check all that apply. write important points. help me asap Your clients, spouses Jared and Joaquim, are both 33 years old. They are concerned about saving money for a down payment on a home. Currently they have $120,000 set aside for that purpose. They plan to start a family in about a year and would like to buy the home in six months. The amount they have saved is more than 20% of the value of the homes in the area they want to move. They look to you for a place to save the money until they need it. Which of the following is the most suitable recommendation for the couple?A) A 26-week T-billB) The Windmill Growth FundC) The Oldtime Life Insurance Company Variable AnnuityD) The Benji Money Market fund